¿Cambia la fuerza entre dos cargas puntuales cuando cambia el marco de referencia inercial?

Las dos cargas puntuales son las siguientes:

ingrese la descripción de la imagen aquí

Entonces la fuerza de Coulomb entre ellos es:

F 1 = q 2 4 π ϵ 0 r 2

Cuando el marco de referencia inercial se mueve hacia la izquierda en v (o las cargas puntuales se mueven a la derecha en v ), entonces habrá campo magnético según la ley de Biot-Savart.

La fuerza de Lorenz entre los dos puntos de carga en la dirección vertical será distinta de cero. La magnitud de la fuerza de Lorentz es:

F L = m 0 q 2 v 2 4 π r 2

En wiki sobre la fuerza de Lorentz , dice que:

Los campos eléctrico y magnético dependen de la velocidad de un observador, por lo que la forma relativista de la ley de fuerza de Lorentz se puede exhibir mejor a partir de una expresión independiente de las coordenadas para los campos electromagnético y magnético,[27] \mathcal{F}, y una dirección temporal arbitraria

Considerando F 1 y F L tienen direcciones y velocidades ópticas opuestas 1 / C 2 = m 0 ϵ 0 , la magnitud de la fuerza F 2 entre las dos cargas se convierte en:

F 2 = F 1 F L = ( 1 v 2 C 2 ) F 1

¿Tengo razón?

ingrese la descripción de la imagen aquí

Según el cálculo aquí , es fácil obtener el factor de dilatación del tiempo 1 v 2 C 2 de un evento en el que las dos cargas puntuales se separan a una distancia de r .

La EDO no lineal:

d 2 r ( t ) d t 2 = ( 1 v 2 C 2 ) q 2 2 π metro ϵ 0 ( R + r ( t ) ) 2

con condiciones iniciales/de contorno

r ( t ) = 0 , r ( t ) = 0

Solución:

L mi F t d r = d 2 r d t 2 d r = d ( d r d t ) d r d t = 1 2 d ( ( d r d t ) 2 ) = 1 2 ( d r d t ) 2

R i gramo h t d r = ( 1 v 2 C 2 ) q 2 d r 2 π metro ϵ 0 ( R + r ) 2 = ( 1 v 2 C 2 ) q 2 r 2 π metro ϵ 0 ( R + r ) R

d r d t = ( 1 v 2 C 2 ) q 2 r π metro ϵ 0 ( R + r ) R

d t = π metro ϵ 0 ( R + r ) R ( 1 v 2 C 2 ) q 2 r d r

d t = π metro ϵ 0 ( 1 v 2 C 2 ) q 2 ( R + r ) R r d r

Δ t ( r ) = 1 1 v 2 C 2 π metro ϵ 0 q 2 ( r R ( r + R ) 1 2 R 3 / 2 ( registro R 2 registro ( r + R + r ) ) )

lo cual indica:

El intervalo de tiempo en que las dos cargas puntuales se separan una distancia mayor r en el marco de referencia móvil es 1 1 v 2 C 2 que en el marco de referencia estático donde v = 0 .

¿Significa esto que la transformación de Lorentz para el tiempo es natural para un evento que involucra leyes tanto eléctricas como magnéticas sin la necesidad de considerar la relatividad especial?

¿Se puede derivar una versión completa de la transformación de Lorentz en tal enfoque?

PD

Encontré que esta pregunta sin una respuesta aceptada es similar, pero hice algunos cálculos y extensiones adicionales aquí.

LCFactorización: Si r (la separación entre A y B ) está cambiando mientras v (la velocidad a la que ambos A y B se están moviendo wrt. un sistema de referencia adecuado) permanece constante, entonces eso es... difícil de comprender para mí. En cuanto a la cuestión implícita más amplia de si las relaciones geométricas pueden derivarse de las " leyes " de la (electro)dinámica: formalmente, tal vez, sí; AFAIU. Sin embargo, al abordar el problema experimentalmente, siempre necesitamos determinar primero las relaciones geométricas, y por separado, para averiguar en primer lugar, por ejemplo, si A 'arena B Los cargos de eran iguales (como usted supuso).
La fuerza de Coulomb no viene dada por F 1 en el cuadro en movimiento. El campo eléctrico es diferente.

Respuestas (3)

¿Significa esto que la transformación de Lorentz para el tiempo es natural para un evento que involucra leyes tanto eléctricas como magnéticas sin la necesidad de considerar la relatividad especial?

Significa que las transformaciones de Lorentz se derivan de las leyes eléctricas y magnéticas. Así es como se descubrió la relatividad especial en primer lugar.

Su derivación parece correcta. Las ecuaciones de Maxwell no son invariantes bajo las transformaciones newtonianas, solo bajo las transformaciones de Lorentz.

Yo respondería "sí". La fuerza de Coulumb q mi cambia, y he aquí por qué: En el marco en reposo B=0, mientras que en el marco en movimiento (') B' no es cero. Desde mi 2 B 2 es invariante de Lorentz: E 'debe ser mayor que E. (Que es el aplastamiento relativista familiar de campos, como en un detector de radiación de transición).

La respuesta es no , la fuerza de Coulomb entre dos cargas puntuales no cambia, a menos que cambies la distancia entre ellas. Si las cargas se mueven en paralelo , la ecuación de Coulomb ya no se aplica. En su lugar, debe usar la ecuación de Lorentz. En otras palabras, si el marco de referencia se mueve y las cargas están estacionarias, se aplica la ecuación de Coulomb. Si el marco de referencia es estacionario y las cargas se mueven, se aplica la ecuación de Lorentz. ¡No puedes aplicar ambas ecuaciones al mismo tiempo!

La pregunta era si la fuerza cambió, no si cambió la fuerza de Coulomb. La fuerza neta viene dada siempre por la fuerza de Lorentz y no es invariante de marco.